−(−49) = −49 true or false?

Answers

Answer 1
False.

Whenever you see a negative sign next to another negative sign, you will always get a positive. So -(-49) is equal to +49
With that information we can determine that it is not equal to negative 49.

Related Questions

Musah stands at the centre of a rectangular field. He first takes 50 steps north, then 25 steps

west and finally 50 steps on a bearing of 3150

.

i. Sketch Musah’s movement

ii. How far west is Musah’s final point from the centre?
iii. How far north is Musah’s final point from the centre?

iv. Describe how you would guide a JHS student to find the bearing and distance of

Musah’s final point from the centre. ​

Answers

Answer:

ii. 75 steps

iii. 75 steps

iv. 106 steps, and [tex]315^{0}[/tex]

Step-by-step explanation:

Let Musah's starting point be A, his waiting point after taking 50 steps northward and 25 steps westward be B, and his stopping point be C.

ii. From the second attachment, Musah's distance due west from A to C (AD) can be determined as;

bearing at B = [tex]315^{0}[/tex], therefore <BCD = [tex]45^{0}[/tex]

To determine distance AB,

[tex]/AB/^{2}[/tex] = [tex]/50/^{2}[/tex]   +  [tex]/25/^{2}[/tex]

          = 25000 + 625

          = 3125

AB = [tex]\sqrt{3125}[/tex]

     = 55.90

AB ≅ 56 steps

Thus, AC = 50 steps + 56 steps

               = 106 steps

From ΔACD,

Sin [tex]45^{0}[/tex] = [tex]\frac{x}{106}[/tex]

⇒ x = 106 × Sin [tex]45^{0}[/tex]

      = 74.9533

     ≅ 75 steps

Musah's distance west from centre to final point is 75 steps

iii. From the secon attachment, Musah's distance north, y, can be determined by;

Cos [tex]45^{0}[/tex] = [tex]\frac{y}{106}[/tex]

⇒ y = 106 × Cos [tex]45^{0}[/tex]

      = 74.9533

      ≅ 75 steps

Musah's distance north from centre to final point is 75 steps.

iv. Musah's distance from centre to final point is AC = AB + BC

                                     = 50 steps + 56 steps

                                     = 106 steps

From ΔACD,

Tan θ = [tex]\frac{75}{75}[/tex]

          = 1.0

θ = [tex]Tan^{-1}[/tex]  1.0

 = [tex]45^{0}[/tex]

Musah's bearing from centre to final point = [tex]45^{0}[/tex] + [tex]270^{0}[/tex]

                                                           =  [tex]315^{0}[/tex]

Karl needs a total of $30 to buy a bike. He has $12. He can earn $6 an hour
babysitting. Which equation can be used to find the number of hours, h, Karl has to
babysit to have the money he needs?

30 - 6h + 12 = 0
6+ n = 12
6 + 12 h = 30
6 h + 12 = 30​

Answers

Answer:

6h + 12 = 30

Step-by-step explanation:

Hence, the equation obtained for number of hours worked is given as  12 + 6h = 30.

How to write a linear equation?

A linear equation for the given case can be written by assuming any variable as the unknown quantity. Then, as per the given data the required operations are done and it is equated to some value.

The total money required is given as $30.

Suppose the number of hours for babysitting be h.

Then, the money earned by doing it is $6h.

And, the total money with Karl is 12 + 6h.

As per the question, the following equations can be written as,

12 + 6h = 30

Hence, the equation for finding the number of hours is given as 12 + 6h = 30.

To know more about linear equation click on,

https://brainly.com/question/11897796

#SPJ2

Find the maximum and minimum values of the function f(x,y)=2x2+3y2−4x−5 on the domain x2+y2≤100. The maximum value of f(x,y) is:

Answers

First find the critical points of f :

[tex]f(x,y)=2x^2+3y^2-4x-5=2(x-1)^2+3y^2-7[/tex]

[tex]\dfrac{\partial f}{\partial x}=2(x-1)=0\implies x=1[/tex]

[tex]\dfrac{\partial f}{\partial y}=6y=0\implies y=0[/tex]

so the point (1, 0) is the only critical point, at which we have

[tex]f(1,0)=-7[/tex]

Next check for critical points along the boundary, which can be found by converting to polar coordinates:

[tex]f(x,y)=f(10\cos t,10\sin t)=g(t)=295-40\cos t-100\cos^2t[/tex]

Find the critical points of g :

[tex]\dfrac{\mathrm dg}{\mathrm dt}=40\sin t+200\sin t\cos t=40\sin t(1+5\cos t)=0[/tex]

[tex]\implies\sin t=0\text{ OR }1+5\cos t=0[/tex]

[tex]\implies t=n\pi\text{ OR }t=\cos^{-1}\left(-\dfrac15\right)+2n\pi\text{ OR }t=-\cos^{-1}\left(-\dfrac15\right)+2n\pi[/tex]

where n is any integer. We get 4 critical points in the interval [0, 2π) at

[tex]t=0\implies f(10,0)=155[/tex]

[tex]t=\cos^{-1}\left(-\dfrac15\right)\implies f(-2,4\sqrt6)=299[/tex]

[tex]t=\pi\implies f(-10,0)=235[/tex]

[tex]t=2\pi-\cos^{-1}\left(-\dfrac15\right)\implies f(-2,-4\sqrt6)=299[/tex]

So f has a minimum of -7 and a maximum of 299.

Decide whether the pair of ratios form a proportion 15/12=4.5/3.6

Answers

Answer: Yes they form a proportion. The given equation is a true equation.

==========================================

Explanation:

The idea is that if we have

a/b = c/d

then that it is the same as

a*d = b*c

This is known as cross multiplication. We'll use this rule to get

15/12 = 4.5/3.6

15*3.6 = 12*4.5

54 = 54

We got the same value on both sides, meaning that the last equation is true. Consequently, it means the first equation is true as well (all three equations are true).

--------

You could also use your calculator to see that

15/12 = 1.25

4.5/3.6 = 1.25

showing that 15/12 = 4.5/3.6 is a true equation and the ratios form a proportion.

Answer:

15/12=4.5/3.6 = True

Step-by-step explanation:

Simplify the following:  Left-hand

15/12

Hint: | Reduce 15/12 to lowest terms. Start by finding the GCD of 15 and 12.

The gcd of 15 and 12 is 3, so 15/12 = (3×5)/(3×4) = 3/3×5/4 = 5/4:

Answer: 5/4

______________________________

Approximate the following:

4.5/3.6

Hint: | Express 4.5/3.6 in decimal form.

4.5/3.6 = 1.25:

Answer:  1.25 = 5/4

Megan has 12 pounds of cheesecake. On Monday, she and her friends eat 4 pounds. On Tuesday, she and her friends eat another 3 pounds. On Wednesday, her friend Mark gives her some more cheesecake so that she has 3 times as much as she had at the end of Tuesday. On Thursday, some of her cheesecake goes bad, so she has the amount that she had at the end of Wednesday, but divided by 5. On Friday, she gives 3 pounds to her dog. On Saturday, her mom gives her one more pound. On Sunday, how many pounds of cheesecake does Megan have left?

Answers

Answer:

Step-by-step explanation:

First we start with 12 pounds

On Monday, she and her friends eat 4 pounds. So we have 8 now.

On Tuesday, she and her friends eat another 3 pounds. So we gave 5 now.

On Wednesday, her friend Mark gives her some more cheesecake so that she has 3 times as much as she had at the end of Tuesday. 5 * 3 = 15

On Thursday, some of her cheesecake goes bad, so she has the amount that she had at the end of Wednesday, but divided by 5. She had 15 at the end of Wednesday. 15/5 = 3.

On Friday, she gives 3 pounds to her dog. 5 - 3 = 2.

On Saturday, her mom gives her one more pound. 2 + 1 = 3.

On Sunday, she finally has 3 pounds.

Answer:

nnnnnnnnnnnnnnnnnnnnnnnnnnnnnnnnnnnnnnnnnnnnnnnnnnnnnn

Step-by-step explanation:

A local mattress manufacturer wants to know if its manufacturing process is in or out of control and has hired you, a statistics expert in the field, to analyze its process. Specifically, the business has run 20 random samples of size 5 over the past month and has determined the mean of each sample.
a. Determine the estimate of the mean when the process is in control.
b. Assuming the process standard deviation is .50 and the mean of the process is the estimate calculated in part a, determine the Upper Control Limit (UCL) and the Lower Control Limit (LCL) for the manufacturing process.
c. Explain the results to the vice-president of the mattress manufacturer focusing on whether, based on the results, the process is in or out of control.
Sample no. Mean of Sample
1 95.72
2 95.44
3 95.40
4 95.50
5 95.56
6 95.72
7 95.60
8 95.24
9 95.46
10 95.44
11 95.80
12 95.20
13 94.82
14 95.78
15 95.18
16 95.32
17 95.08
18 95.22
19 95.04
20 95.

Answers

Answer:

Answer to question a = 95.4

Answer to question b = UCL = 96.07

LCL = 94.73

Answer to question c = Process is still in control

Step-by-step explanation:

a. The computation of estimate mean is as shown below:-

= 95.4

b. The computation of Upper Control Limit (UCL) and the Lower Control Limit (LCL) for the manufacturing process is shown below:-

= 95.4 + 0.67082

= 96.07

= 95.4 - 0.67082

= 94.73

c. The explanation is shown below:-

From the above calculation we can see that the sample lies between LCL AND UCL that is (94.73 ,96.07) ,

The Process is still in control

5/3 x 6/7 real quick plz

Answers

Answer:

10/7 or 1 3/7. I hope this helps,

Step-by-step explanation:

Hi people if someone gives me a hint please. Show algebraically that the product of two consecutive numbers is always even l wrote n (n+1) is always an even number But doesnt recognise it as 100% right thanks for any help

Answers

Step-by-step explanation:

Consider the following rules.

even + odd = odd

even - odd = odd

even × odd = even

even ÷ odd = even (if divisible)

Now for the two consectives terms...

One will surely be even and the other, odd.

So using the rule

Their product will always be odd

Hope it helps....

BRAINLIEST PRETTY PLEASE!!

If you use a 5/8 inch drill bit instead of a 3/16 that the project called for ,your hole will be too . by inches

Answers

5/8 - 3/16
= 10/16 - 3/16
= 7/16
Therefore the hole will too big by 7/16 of an inch.

Suppose we want to test the color distribution claim on the M&M’s website that a bag of plain M&M’s is made up of 10% blue, 10% orange, 10% green, 20% red, 20% yellow, and 30% brown. We select a sample of 400 plain M&M’s and found the following: Color Blue Orange Green Red Yellow Brown Frequency 30 48 55 66 70 131
Is there evidence to doubt the color distribution claimed by the website? Use =0.05

Answers

Answer:

Calculated χ² = 13.425

χ² (5,0.025) >14.45 and χ²(5,0.975) <1.24

The given data does not fall in the critical region so we accept H0 and conclude there is no evidence to doubt the color distribution claimed by the website.

Step-by-step explanation:

Color             Blue      Orange     Green    Red   Yellow    Brown

Frequency     30         48              55        66         70         131

Expected      40           40              40        80          80        120

H0:  The bag of plain M&Ms is made up of 10% blue, 10% orange, 10% green, 20% red, 20% yellow, and 30% brown

Ha: The color distribution is not equal to  the distribution stated in the null hypothesis.

Calculate chi square

χ² = (30-40)² /40 + (48-40)²/40 + (55-40)²/40 + (66-80)²/80 + (70-80)²/80 + (131-120)²/120

χ² = 2.5 + 1.6 + 5.625 + 2.45 + 1.25= 13.425

The critical region for χ²  for 5 degrees of freedom with ∝= 0.05 is

χ² (5,0.025) >14.45 and χ²(5,0.975) <1.24

The given data does not fall in the critical region so we accept H0 and conclude there is no evidence to doubt the color distribution claimed by the website.

What is the answer, what are the steps to solve this, and what do the parts of the equation represent?

Answers

Step-by-step explanation:

Just sub 4 into where n is

How to evaluate this help me out so lost?

Answers

Answer:

5443

Step-by-step explanation:

Order of Operations: BPEMDAS

Always left to right.

Step 1: Add 68 and 5042

68 + 5042 = 5110

Step 2: Add 5110 and 333

5110 + 333 = 5443

And we have our answer!

Last question of the day!!

Answers

Answer:

Correct options are 2, 5 and 7.

Step-by-step explanation:

Consider the given vertices of triangle are A(-3,-3), B(-3,2) and C(1,2).

Distance formula:

[tex]d=\sqrt{(x_2-x_1)^2+(y_2-y_1)^2}[/tex]

Using distance formula, we get

[tex]AB=\sqrt{(-3-(-3))^2+(2-(-3))^2}[/tex]

[tex]AB=\sqrt{(0)^2+(5)^2}[/tex]

[tex]AB=\sqrt{25}[/tex]

[tex]AB=5[/tex]

Similarly,

[tex]BC=\sqrt{(1-(-3))^2+(2-2)^2}=4[/tex]

[tex]AC=\sqrt{(1-(-3))^2+(2-(-3))^2}=\sqrt{16+25}=\sqrt{41}[/tex]

From the above calculation it is clear that AC>AB and AC>BC.

According to Pythagoras theorem, in a right angle triangle, the square of largest side is equal to the sum of squares of two small sides.

[tex]hypotenuse^2=base^2+perpendicular^2[/tex]

[tex]AC^2=(\sqrt{41})^2=41[/tex]

[tex]AB^2+BC^2=(5)^2+4^2=24+16=41=AC^2[/tex]

So, given triangle is a right angle triangle and AC is its hypotenuse.

Therefore, the correct options are 2, 5 and 7.

Find the particular solution of the differential equation that satisfies the initial condition(s). (Remember to use absolute values where appropriate.) f ''(x) = 4 x2 , f '(1) = 2, f(1) = 5

Answers

Looks like either [tex]f''(x)=4x^2[/tex] or [tex]f''(x)=\frac4{x^2}[/tex]...

In the first case, integrate both sides twice to get

[tex]f''(x)=4x^2\implies f'(x)=\dfrac43x^3+C_1\implies f(x)=\dfrac13x^4+C_1x+C_2[/tex]

Then the initial conditions give

[tex]f'(1)=2\implies 2=\dfrac43\cdot1^3+C_1\implies C_1=\dfrac23[/tex]

[tex]f(1)=5\implies 5=\dfrac13\cdot1^4+C_1\cdot1+C_2\implies C_2=4[/tex]

so that the particular solution is

[tex]f(x)=\dfrac{x^4}3+\dfrac{2x}3+4[/tex]

If instead [tex]f''(x)=\frac4{x^2}[/tex], we have

[tex]f''(x)=\dfrac4{x^2}\implies f'(x)=-\dfrac4x+C_1\implies f(x)=-4\ln|x|+C_1x+C_2[/tex]

[tex]f'(1)=2\implies 2=-\dfrac41+C_1\implies C_1=6[/tex]

[tex]f(1)=5\implies 5=-4\ln|1|+C_1\cdot1+C_2\implies C_2=-1[/tex]

[tex]\implies f(x)=-4\ln|x|+6x-1[/tex]

Combine like terms. What is a simpler form of each expression? 4c-4d+8c-3d

Answers

Answer:

12c-7d

Step-by-step explanation:

[tex]4c-4d+8c-3d=0\\4c+8c=3d+4d\\12c=7d\\12c-7d[/tex]

Answer: 12c-7d

===============================================

Explanation:

The terms 4c and 8c are one pair of like terms that combine to 4c+8c = 12c. We add 4 and 8 to get 12, then tack a 'c' at the end

The other pair of like terms are -4d and -3d. They combine to -7d for similar reasoning.

12c and -7d are not like terms, so we can't combine them and we stop here.

-----------

One way to think of combining like terms is consider simplifying 2c+3c. You could say that 2c represents having 2 cups while 3c is having 3 cups. Writing 2c+3c means we start with 2 cups and add on 3 more getting a total of 2+3 = 5 cups. Symbolically we would then write 5c. Therefore 2c+3c = 5c.

[tex]f(x) = sqr root x+3 ; g(x) = 8x - 7[/tex]

Find (f(g(x))

Answers

[tex]f(x)=\sqrt{x+3}\\g(x)=8x-7\\\\f(g(x))=\sqrt{8x-7+3}=\sqrt{8x-4}[/tex]

Can someone do this assuming that it is infinite and as well as assuming it's not infinite? Thanks!

Answers

Answer:

see below

Step-by-step explanation:

4,7,12,19

We are adding 3,5,7,9..... each time

The sequence is not arithmetic because we are not adding a constant.  It is not geometric since we are not multiplying by a constant term each time

There is no common difference  or common ratio.

The explicit formula is

an =n^2 +3

 

The recursive formula is

(n+1)^2 +3 - (n^2 +3)

    n^2 +2n+1+3 - ( n^2+3)

      2n+1

a sub(n+1) = a sub( n) + 2n+1

The 10th term

an      = n^2 +3

Let n=10

an = 10^2+3

    = 100+3

    = 103

summation

see image

since the numbers are increasing and greater than 1 the sum does not exist

what should be added to 66.778 get 78.2​

Answers

Answer:

11.422

Step-by-step explanation:

[tex]78.2 - 66.778 \\ = 11.422[/tex]

The chart shows a certain city's population by age. Assume that the selections are independent events. If 8 residents of this city are selected at random, find the probability that the first 2 are 65 or older, the next 3 are 25-44 years old, the next 2 are 24 or younger, and the last is 45-64 years old.

Answers

Answer:

0.000014

Step-by-step explanation:

The chart is not provided so i will use an example chart to explain the answer. Here is a sample chart:

City X's Population by Age

0-24 years old 33%

25-44 years old 22%

45-64 years old 21%

65 or older 24%

In order to find probability of independent events we find the probability of each event occurring separately and then multiply the calculated probabilities together in the following way:

P(A and B) = P(A) * P(B)

probability that the first 2 are 65 or older

Let A be the event that the first 2 are 65 or older

The probability of 65 or older 24% i.e. 0.24

So the probability that first 2 are 65 or older is:

0.24(select resident 1) * 0.24(select resident 2)

P(A) = 0.24 * 0.24

       = 0.0576

P(A) = 0.0576

probability that the next 3 are 25-44 years old

Let B be the event that the next 3 are 25-44 years old

25-44 years old 22%  i.e. 0.22

So the probability that the next 3 are 25-44 years old is:

0.22 * 0.22* 0.22

P(B) = 0.22 * 0.22 * 0.22

      = 0.010648

P(B) = 0.010648

probability that next 2 are 24 or younger

Let C be the event that the next 2 are 24 or younger

0-24 years old 33% i.e. 0.33

So the probability that the next 2 are 24 or younger is:

0.33 * 0.33

P(C) = 0.33 * 0.33

       = 0.1089

P(C) = 0.1089

probability that last is 45-64 years old

Let D be the event that last is 45-64 years old

45-64 years old 21%  i.e. 0.21

So the probability that last is 45-64 years old is:

0.21

P(D) = 0.21

So probability of these independent events is computed as:

P(A and B and C and D) = P(A) * P(B) * P(C) * P(C)

                                        = 0.0576 * 0.010648  * 0.1089  * 0.21

                                        = 0.000014

The expression −50x+100 represents the balance, in dollars, of a bank account after x months. What is the rate of change, in dollars per month, of the bank account balance?

Answers

Answer:

-50

Step-by-step explanation:

Basically get two slopes -50(1)+100 will get you 1,50 (1 is x and 50 is y since its the answer)

-50(0)+100 (0,100)  Y₂-Y₁/X₂-X₁ 50-100/1-0

Rate of change per month = -$50

At a sale, dresses were sold for $39 each. This price was 65% of a dress's original price. How much did a dress originally cost?

Answers

Answer:

Hey there!

We can write the equation:

0.65x=39

x=60

The dress originally sold for 60 dollars.

Hope this helps :)

Cancel the common factor of the numerator and the denominator and write specified expression

Answers

Step-by-step explanation:

Hello,

I hope you mean to cancel the common factor that exists in numerator and denominator,right.

so, Let's look for the common factor,

here, the expression is,

=4(x-2)/ (x+5)(x-2)

so, here we find the common factor is (x-2)

now, we have to cancel it. And after cancelling we get,

=4/(x+5)

Note:{ we cancel the common factor if the common factors are in multiply form.}

Hope it helps

Consider F and C below.
F(x, y) = x2 i + y2 j
C is the arc of the parabola y = 2x2 from (−1, 2) to (2, 8)
(a) Find a function f such that F = ∇f. f(x, y) =
(b) Use part (a) to evaluate C ∇f · dr along the given curve C.

Answers

(a)

[tex]\dfrac{\partial f}{\partial x}=x^2\implies f(x,y)=\dfrac{x^3}3+g(y)[/tex]

[tex]\dfrac{\partial f}{\partial y}=\dfrac{\mathrm dg}{\mathrm dy}=y^2\implies g(y)=\dfrac{y^3}3+C[/tex]

[tex]\implies f(x,y)=\dfrac{x^3+y^3}3+C[/tex]

(b)

[tex]\displaystyle\int_C\nabla f\cdot\mathrm d\mathbf r=f(2,8)-f(-1,2)=\boxed{171}[/tex]

Assume that thermometer readings are normally distributed with a mean of 0C and a standard deviation of 1.00C. A thermometer is randomly selected and tested. For the case​ below, draw a​ sketch, and find the probability of the reading.​ (The given values are in Celsius​ degrees.) Between and

Answers

Answer: 0.0546 and 0.9829

Step-by-step explanation:

solution:

= P( 1.50< Z <2.25 )

= P(Z <2.25 ) - P(Z <1.50 )

Using z table,

= 0.9878-0.9332

=0.0546

b.

= P( -2.12< Z <3.73 )

= P(Z <3.73) - P(Z <-2.12 )

Using z table,

= 0.9999-0.0170

=0.9829

Plz answer last question and im lost!

Answers

Answer:

[tex]\pi[/tex] radian

Step-by-step explanation:

We know that angle for a full circle is 2[tex]\pi[/tex]

In the given figure shape is semicircle

hence,

angle for semicircle will be half of angle of full circle

thus, angle for given figure = half of  angle for a full circle = 1/2 * 2[tex]\pi[/tex] = [tex]\pi[/tex]

Thus, answer is [tex]\pi[/tex] radian

alternatively, we also know that angle for a straight line is 180 degrees

and 180 degrees is same as [tex]\pi[/tex] radian.

A recent study of the relationship between social activity and education for a sample of corporate executives showed the following results. Social Activity Education Above Average Average Below Average College 30 20 10 High School 20 40 90 Grade School 10 50 130 Using 0.05 as the significance level, what is the critical value for the test statistic

Answers

Answer:

9.488

Step-by-step explanation:

The critical value is found by first assessing which statistical test should be used.

We are interested in investigating relationship between social activity and education so chi-square test would be appropriate.

We have 3 rows and 3 columns. The degree of freedom for chi-square critical value is (r-1)(c-1)=(3-1)(3-1)=2*2=4

Chi-square critical value(0.05,4)= 9.488

The head of a computer science department is interested in estimating the proportion of students entering the department who will choose the new computer engineering option. Suppose there is not information about the proportion of students who might choose the option. What size sample should the department head take if he wants to be 95% confident that the estimate is within 0.10 of the true proportion

Answers

Answer:

96

Step-by-step explanation:

From the given information:

At 95% Confidence interval level,Level of significance [tex]\alpha[/tex] 0.05, the value of Z from the standard normal tables = 1.96

Margin of Error = 0.10

Let assume that the estimated proportion = 0.5

therefore; the sample size n can be determined by using the formula: [tex]n =(\dfrac{Z}{E})^2 \times p\times (1-p)[/tex]

[tex]n =(\dfrac{1.96}{0.1})^2 \times 0.5\times (1-0.5)[/tex]

[tex]n =(19.6)^2 \times 0.5\times (0.5)[/tex]

n = 96.04

n [tex]\approx[/tex] 96

the difference of 8 and 2, added to x"

Answers

Answer:

see below

Step-by-step explanation:

Difference is subtract

(8-2)

Then add this to x

(8-2) +x

6+x

Given: x - 5 > -2. Choose the solution set.

Answers

Answer: x>3

Step-by-step explanation:

x-5>2

x>+5-2

x>3

Select the function that represents a parabola with zeros at x = –2 and x = 4, and y-intercept (0,–16). A ƒ(x) = x2 + 2x – 8 B ƒ(x) = 2x2 + 4x – 16 C ƒ(x) = x2 – 2x – 8 D ƒ(x) = 2x2 – 4x – 16

Answers

Answer:

D. [tex]f(x) = 2\cdot x^{2}-4\cdot x -16[/tex]

Step-by-step explanation:

Any parabola is modelled by a second-order polynomial, whose standard form is:

[tex]y = a\cdot x^{2}+b\cdot x + c[/tex]

Where:

[tex]x[/tex] - Independent variable, dimensionless.

[tex]y[/tex] - Dependent variable, dimensionless.

[tex]a[/tex], [tex]b[/tex], [tex]c[/tex] - Coefficients, dimensionless.

In addition, a system of three linear equations is constructed by using all known inputs:

(-2, 0)

[tex]4\cdot a -2\cdot b + c = 0[/tex] (Eq. 1)

(4, 0)

[tex]16\cdot a + 4\cdot b +c = 0[/tex] (Eq. 2)

(0,-16)

[tex]c = -16[/tex] (Eq. 3)

Then,

[tex]4\cdot a - 2\cdot b = 16[/tex] (Eq. 4)

[tex]16\cdot a + 4\cdot b = 16[/tex] (Eq. 5)

(Eq. 3 in Eqs. 1 - 2)

[tex]a - 0.5\cdot b = 4[/tex] By Eq. 4 (Eq. 4b)

[tex]a = 4 + 0.5\cdot b[/tex]

Then,

[tex]16\cdot (4+0.5\cdot b) + 4\cdot b = 16[/tex] (Eq. 4b in Eq. 5)

[tex]64 + 12\cdot b = 16[/tex]

[tex]12\cdot b = -48[/tex]

[tex]b = -4[/tex]

The remaining coeffcient is:

[tex]a = 4 + 0.5\cdot b[/tex]

[tex]a = 4 + 0.5\cdot (-4)[/tex]

[tex]a = 2[/tex]

The function that represents a parabola with zeroes at x = -2 and x = 4 and y-intercept (0,16) is [tex]f(x) = 2\cdot x^{2}-4\cdot x -16[/tex]. Thus, the right answer is D.

Answer:

D ƒ(x) = 2x2 – 4x – 16

Step-by-step explanation:

Other Questions
in a 10 team league, each teams play every other team exactly twice. find the total number of games played in the league The bowling scores for six people are:27, 142, 145, 146, 154, 162What is the most appropriate measure of center?O A. The standard deviationO B. The rangeO C. The medianO D. The mean El abuelo ____________ a la playa durante los veranos. a. iba b. iban c. ibas Please select the best answer from the choices provided A B C How were U.S government and society affected by the Vietnam War? The first battle of the Spanish-American War was fought in A) Havana, Cuba B) the Philippines C) Miami, Florida D) Puerto Rico I NEED HELP NOW PLZ According to the CDC what are health determinants? A. Factors that contribute to a persons current state of healthB. The aspects of a persons physical environmentC. Health factors that can be controlledD. Health factors that cannot be controlled what is the domain of this According to the United States Golf Association, the diameter of a golf ball should not be less than 42.67 millimeters. What is the estimate of this value rounded to the nearest tenth of a millimeter? Read the excerpt from "Flight into Yesterday." This leg was also the most dangerous of the entire trip. Howland, 2,600 miles to the east, was a tiny speck of land in the middle of the vast Pacific. It would take all of her skill as a pilot and Freds as a navigator to find the island. But danger did not worry her. She knew that flying carried risks. She faced them and accepted them. What mattered was setting herself a challenge and meeting it. What mattered was showing that women could do what men could do and encouraging other women to do what they were capable of. That was why she was flying over the Pacific, looking for a speck of land. Fame was never her goal. The important thing was to do what she had set out to do and to do it as well as she knew how. She had been that way all her life. Amelia Earhart, world-famous flier, was very much like the young girl who once lived in Atchison, Kansas. What best describes the authors purpose? (Economics) You have two friends who are pursuing different careers after high school this year. Danielle is interested in getting into the medical field and wants to be a doctor after eight years of schooling. Danny wants to be a barber after he earns a certificate after six months of barber school. 1)Predict: How will supply and demand and labor substitutes in these job markets affect their pay and likeliness to get a job?2)How will supply and demand within the job market affect my career choice? Click on the box to choose whether the sentence is correct (C), contains a comma splice (CS), or is a fused sentence(FS)Don't stack twelve boxes on top of one another, it's the twelfth one that will knock down the pile. Which of the following processes is exothermic? Group of answer choices Allowing meat to thaw after taking it out of the freezer Popsicle melting on a warm summer day Boiling water in a beaker to convert it to steam Rolling a ball up a hill Reacting hydrogen and oxygen gases to make water f(x)=5x^34x^2+8x and g(x)=4x^2+8, find (fg)(x) and (fg)(2). Evaluate fx, fy, fz at the given point a) f (x, y z) = xyz at the point (1, 2, 3) b) f (x, y, z) = x - 2xy + 3yxz at the point (3, 1, -2) I need help im not understanding what to do. For this assignment, you'll be creating a document or project. Now that you have an idea of the organ systems in the human body, its time to choose one that interests you the most. Use the links below to find detailed information about the organ system you choose. You will use this information to create a poster in Glogster or PowerPoint that illustrates the major facts and details of the system you chose. To make it fun and interesting, create a Travel Guide that takes the reader through the system, detailing the organs in that system and what they do for the body. To complete this assignment follow these steps: Choose your organ system to profile. Research the following information about your organ system: Short description of the organ system: What does the organ system do? (IE: endocrine system) How is the system organized? What are the major organs in this system? What is the main or most important organ in the system? How do these organs work together to do what they do? What are the functions in the human body that are done by this organ system? What would happen to the body if this organ system failed, or didnt work right? Use the links to websites to gather that information. Save any graphics you want to use in creating your Poster. If you choose Glogster: Go to Glogster and follow their instructions in setting up an account. Its easy and its free. Watch the tutorial and then get started! A multiple choice test contains 10 questions with 5 answer choices. What is the probability of correctly answering 5questions if you guess randomly on each question? Given the following venn diagram, choose the correct set for M Gene is playing a game with a bag of marbles. 3 of the marbles are blue, 4 are green, and 7 are yellow. See below for awarded prizes. $2 green $0.5 yellow $4 blue What is the expected cost (or payout for Gene's game? The graphic below shows a protractor, which is used to measure angles. What is the level of accuracy of this measurement tool? its a projractor All the following statements concerning universal life insurance are correct EXCEPT: Group of answer choices Interest credited to the cash value is taxable to a policyowner in the year credited. The policyowner can add to a policy's cash value at any time subject to policy guidelines. Interest is credited to the policy's cash value each month. Withdrawals from a policy's cash value reduce the death benefit.